Respiratory Ch20-24 PrepU

Pataasin ang iyong marka sa homework at exams ngayon gamit ang Quizwiz!

A nursing student knows that there are three most common symptoms of asthma. Select all that apply. a) Cough b) Crackles c) Dyspnea d) Wheezing

A, B, D. The three most common symptoms of asthma are cough, dyspnea, and wheezing. In some instances, cough may be the only symptom.

The nurse is assessing a patient for obstructive sleep apnea (OSA). Which of the following are signs and symptoms of OSA? Select all that apply. a) Insomnia b) Polycythemia c) Evening headaches d) Loud snoring e) Pulmonary hypotension

A, B, D. Signs and symptoms include excessive daytime sleepiness, frequent nocturnal awakening, insomnia, loud snoring, morning headaches, intellectual deterioration, personality changes, irritability, impotence, systemic hypertension, dysrhythmias, pulmonary hypertension, corpulmonale, polycythemia, and enuresis.

Following are statements regarding medications taken by a patient diagnosed with COPD. Choose which statements correctly match the drug name to the drug category. Select all that apply. a) Prednisone is a corticosteroid. b) Ciprofloxacin is an antibiotic. c) Bactrim is a bronchodilator. d) Decadron is an antibiotic. e) Albuterol is a bronchodilator.

A, B, E. Theophylline, albuterol, and atropine are bronchodilators. Dexamethasone (Decadron) and prednisone are corticosteroids. Amoxicillin, ciprofloxacin, and cotrimoxazole (Bactrim) are antibiotics. These are all drugs that could be prescribed to a patient with COPD

Upon assessment, the nurse suspects that a patient with COPD may have bronchospasm. What manifestations validate the nurse's concern? (Select all that apply.) a) Wheezes b) Ascites c) Decreased airflow d) Compromised gas exchange e) Jugular vein distention

A, C, D. Bronchospasm, which occurs in many pulmonary diseases, reduces the caliber of the small bronchi and may cause dyspnea, static secretions, and infection. Bronchospasm can sometimes be detected on auscultation with a stethoscope when wheezing or diminished breath sounds are heard. Increased mucus production, along with decreased mucociliary action, contributes to further reduction in the caliber of the bronchi and results in decreased airflow and decreased gas exchange. This is further aggravated by the loss of lung elasticity that occurs with COPD (GOLD, 2010).

A client with chronic bronchitis is admitted with an exacerbation of symptoms. During the nursing assessment, the nurse will expect which of the following findings? Select all that apply. a) Purulent sputum with frequent coughing b) Respiratory rate of 10 breaths per minute c) Use of accessory muscles to breathe d) Tympany percussed bilaterally over the lung bases e) Hypoventilatory breathing pattern

A, C. Chronic bronchitis increases airway resistance and can thicken bronchial mucosa during an exacerbation. The client will have dyspnea requiring the use of accessory muscles to breathe, along with tachypnea and sputum production. Bronchial irritation and the need to expectorate mucus will lead to coughing. Percussion in this client would lead to resonant or hyperresonant sounds

A patient comes to the clinic with a cold and wants something to help relieve the symptoms. What should the nurse include in educating the patient about the uncomplicated common cold? (Select all that apply.) a) Inform the patient about the symptoms of secondary infection. b) Inform the patient that taking an antihistamine will help to decrease the duration of the cold. c) Tell the patient to take prescribed antibiotics to decrease the severity of symptoms. d) Suggest that the patient take adequate fluids and get plenty of rest. e) Inform the patient that the virus is contagious for 2 days before symptoms appear and during the first part of the symptomatic phase.

A, D, E. Colds are highly contagious because virus is shed for about 2 days before the symptoms appear and during the first part of the symptomatic phase. Management consists of symptomatic therapy that includes adequate fluid intake, rest, prevention of chilling, and the use of expectorants as needed. The nurse instructs the patient about methods to treat symptoms of the common cold and provides both verbal and written information to assist in the prevention and management of URIs

A nurse caring for a patient with a pulmonary embolism understands that a high ventilation-perfusion ratio may exist. What does this mean for the patient? a) Ventilation exceeds perfusion. b) Perfusion exceeds ventilation. c) Ventilation matches perfusion. d) There is an absence of perfusion and ventilation.

A. A high ventilation-perfusion rate means that ventilation exceeds perfusion, causing dead space. The alveoli do not have an adequate blood supply for gas exchange to occur. This is characteristic of a variety of disorders, including pulmonary emboli, pulmonary infarction, and cardiogenic shock

The nurse auscultates the lung sounds of a patient during a routine assessment. The sounds produced are harsh and cracking, sounding like two pieces of leather being rubbed together. The nurse would be correct in documenting this finding as which of the following? a) Pleural friction rub b) Crackles c) Sonorous wheezes d) Sibilant wheezes

A. A pleural friction rub is heard secondary to inflammation and loss of lubricating pleural fluid. Crackles are soft, high-pitched, discontinuous popping sounds that occur during inspiration. Sonorous wheezes are deep, low-pitched rumbling sounds heard primarily during expiration. Sibilant wheezes are continuous, musical, high-pitched, whistlelike sounds heard during inspiration and expiration.

The nurse identifies which finding to be most consistent prior to the onset of acute respiratory distress? a) Normal lung function b) Chronic lung disease c) Loss of lung function d) Slow onset of symptoms

A. Acute respiratory failure occurs suddenly in clients who previously had normal lung function.

The nurse is caring for a patient with recurrent hemoptysis who has undergone a bronchoscopy. Immediately following the procedure, the nurse should complete which of the following? a) Assess the patient for a cough reflex. b) Ensure the patient remains moderately sedated to decrease anxiety. c) Offer the patient ice chips. d) Instruct the patient that bed rest must be maintained for 2 hours.

A. After the procedure, the patient must take nothing by mouth until the cough reflex returns, because the preoperative sedation and local anesthesia impair the protective laryngeal reflex and swallowing. Once the patient demonstrates a cough reflex, the nurse may offer ice chips and eventually fluids. The patient is sedated during the procedure, not afterward. The patient is not required to maintain bed rest following the procedure

Asthma is cause by which type of response? a) IgE-mediated b) IgA-mediated c) IgD-mediated d) IgM-mediated

A. Atopy, the genetic predisposition for the development of an IgE-mediated response to allergens, is the most common identifiable predisposing factor for asthma. Chronic exposure to airway allergens may sensitize IgE antibodies and the cells of the airway

A nurse is preparing a client for bronchoscopy. Which instruction should the nurse give to the client? a) Don't eat. b) Don't walk. c) Don't cough. d) Don't talk.

A. Bronchoscopy involves visualization of the trachea and bronchial tree. To prevent aspiration of stomach contents into the lungs, the nurse should instruct the client not to eat or drink anything for approximately 6 hours before the procedure. It isn't necessary for the client to avoid walking, talking, or coughing

A nurse is aware that crackles, non-contiguous breath sounds, are assessed for a patient with: a) Pulmonary fibrosis b) Asthma c) A collapsed alveoli d) Chronic bronchitis

A. Crackles (formerly referred to as rales) are discrete, noncontinuous sounds that result from delayed reopening of deflated airways. Crackles may or may not be cleared by coughing. They reflect underlying inflammation or congestion and are often present in such conditions as pneumonia, bronchitis, heart failure, bronchiectasis, and pulmonary fibrosis. Crackles are usually heard on inspiration, but they may also be heard on expiration

A nurse is caring for a client who has a tracheostomy and temperature of 103° F (39.4° C). Which intervention will most likely lower the client's arterial blood oxygen saturation? a) Endotracheal suctioning b) Incentive spirometry c) Encouragement of coughing d) Use of a cooling blanke

A. Endotracheal suctioning removes secretions as well as gases from the airway and lowers the arterial oxygen saturation (SaO2) level. Coughing and using an incentive spirometer improve oxygenation and should raise or maintain oxygen saturation. Because of superficial vasoconstriction, using a cooling blanket can lower peripheral oxygen saturation readings, but SaO2 levels wouldn't be affected.

A client has just undergone bronchoscopy. Which nursing assessment is most important at this time? a) Level of consciousness (LOC) b) Memory c) Personality changes d) Intellectual ability

A. Following bronchoscopy, LOC is the most important assessment because changes in the client's LOC may alert the nurse to serious neurologic problems. Memory, personality changes, and intellectual ability are important but don't take precedence at this time.

Which type of oxygen therapy includes the administration of oxygen at pressure greater than 1 atmosphere? a) Hyperbaric b) High-flow systems c) Low-flow systems d) Transtracheal

A. Hyperbaric oxygen therapy is the administration of oxygen at pressures greater than 1 atmosphere. As a result, the amount of oxygen dissolved in plasma is increased, which increases oxygen levels in the tissues. Low-flow systems contribute partially to the inspired gas the patient breathes, which means that the patient breathes some room air along with the oxygen. High-flow systems are indicated for patients who require a constant and precise amount of oxygen. During transtracheal oxygenation, patients achieve adequate oxygenation at lower rates, making this method less expensive and more efficient

A home health nurse is visiting a home care client with advanced lung cancer. Upon assessing the client, the nurse discovers wheezing, bradycardia, and a respiratory rate of 10 breaths/minute. These signs are associated with which condition? A. Hypoxia B. Delirium C. Hyperventilation D. Semiconsciousness

A. Hypoxia. As the respiratory center in the brain becomes depressed, hypoxia occurs, producing wheezing, bradycardia, and a decreased respiratory rate. Delirium is a state of mental confusion characterized by disorientation to time and place. Hyperventilation (respiratory rate greater than that metabolically necessary for gas exchange) is marked by an increased respiratory rate or tidal volume, or both. Semiconsciousness is a state of impaired consciousness characterized by limited motor and verbal responses and decreased orientation.

A nurse is assessing a client who comes to the clinic for care. Which findings in this client suggest bacterial pneumonia? a) Dyspnea and wheezing b) Hemoptysis and dysuria c) Sore throat and abdominal pain d) Nonproductive cough and normal temperature

A. In a client with bacterial pneumonia, retained secretions cause dyspnea, and respiratory tract inflammation causes wheezing. Bacterial pneumonia also produces a productive cough and fever, rather than a nonproductive cough and normal temperature. Sore throat occurs in pharyngitis, not bacterial pneumonia. Abdominal pain is characteristic of a GI disorder, unlike chest pain, which can reflect a respiratory infection such as pneumonia. Hemoptysis and dysuria aren't associated with pneumonia

A patient has had a laryngectomy and was able to retain his airway, with no difficulty swallowing. There is no split of thyroid cartilage. The nurse would record this type of laryngectomy as which of the following? a) Partial laryngectomy b) Supraglottic laryngectomy c) Hemilaryngectomy d) Total laryngectomy

A. In a partial laryngectomy, a portion of the larynx is removed, along with one vocal cord and the tumor; all other structures remain. The airway remains intact, and the patient is expected to have no difficulty swallowing. During a supraglottic laryngectomy, a tracheostomy is left in place until the glottic airway is established. Hemilaryngectomy is done by splitting the thyroid cartilage of the larynx in the midline of the neck, and the portion of the vocal cord is removed with the tumor. During a total laryngectomy, a complete removal of the larynx is performed, including the hyoid bone, epiglottis, cricoids cartilage, and two or three rings of the trachea.

You are caring for a client with chronic respiratory failure. What are the signs and symptoms of chronic respiratory failure? a) Progressive loss of lung function associated with chronic disease b) Sudden loss of lung function associated with chronic disease c) Progressive loss of lung function with history of normal lung function d) Sudden loss of lung function with history of normal lung function

A. In chronic respiratory failure, the loss of lung function is progressive, usually irreversible, and associated with chronic lung disease or other disease. This makes options B, C, and D incorrect

A client appears to be breathing faster than during the last assessment. Which of the following interventions should the nurse perform? a) Count the rate of respirations. b) Inquire if there have been any stressful visitors. c) Assess the radial pulse. d) Assist the client to lie down.

A. Observing the rate and depth of respiration is an important aspect of a nursing assessment. The normal adult resting respiratory rate is 12 to 18 breaths per minute. Tachypnea is rapid breathing with a rate greater than 24 breaths per minute. An increase in the rate of respirations needs further investigation and must be reported.

After diagnosing a client with pulmonary tuberculosis, the physician tells family members that they must receive isoniazid (INH [Laniazid]) as prophylaxis against tuberculosis. The client's daughter asks the nurse how long the drug must be taken. What is the usual duration of prophylactic isoniazid therapy? a) 6 to 12 months b) 2 to 4 months c) 3 to 5 days d) 1 to 3 weeks

A. Prophylactic isoniazid therapy must continue for 6 to 12 months at a daily dosage of 300 mg. Taking the drug for less than 6 months may not provide adequate protection against tuberculosis

A nurse would question the accuracy of a pulse oximetry evaluation in which of the following conditions? a) A client experiencing hypothermia b) A client on a ventilator with PEEP c) A client sitting in a chair after prolonged bed rest d) A client receiving oxygen therapy via Venturi mask

A. Pulse oximetry is a noninvasive method of continuously monitoring the oxygen saturation of hemoglobin. The reading is referred to as SpO2. A probe or sensor is attached to the fingertip, forehead, earlobe, or bridge of the nose. Values less than 85% indicate that the tissues are not receiving enough oxygen. SpO2 values obtained by pulse oximetry are unreliable in states of low perfusion such as hypothermia.

The nurse is caring for a client whose respiratory status has declined since shift report. The client has tachypnea, is restless, and displays cyanosis. Which diagnostic test should be assessed first? a) Pulse oximetry b) Pulmonary function test c) Arterial blood gases d) Chest x-ray

A. Pulse oximetry is a noninvasive method to determine arterial oxygen saturation. Normal values are 95% and above. Using this diagnostic test first provides rapid information of the client's respiratory system. All other options vary in amount of time and patient participation in determining further information regarding the respiratory system.

A client is on a positive-pressure ventilator with a synchronized intermittent mandatory ventilation (SIMV) setting. The ventilator is set for 8 breaths per minute. The client is taking 6 breaths per minute independently. The nurse a) Continues assessing the client's respiratory status frequently b) Contacts the respiratory therapy department to report the ventilator is malfunctioning c) Consults with the physician about removing the client from the ventilator d) Changes the setting on the ventilator to increase breaths to 14 per minute

A. The SIMV setting on a ventilator allows the client to breathe spontaneously with no assistance from the ventilator for those extra breaths. Data in the stem suggest that the ventilator is working correctly. The nurse would continue making frequent respiratory assessments of the client. There are not sufficient data to suggest the client could be removed from the ventilator. There is no reason to increase the ventilator's setting to 14 breaths per minute or to contact respiratory therapy to report the machine is not working properly

A victim has sustained a blunt force trauma to the chest. A pulmonary contusion is suspected. Which of the following clinical manifestations correlate with a moderate pulmonary contusion? a) Blood-tinged sputum b) Productive cough c) Bradypnea d) Respiratory alkalosis

A. The clinical manifestations of pulmonary contusions are based on the severity of bruising and parenchymal involvement. The most common signs and symptoms are crackles, decreased or absent bronchial breath sounds, dyspnea, tachypnea, tachycardia, chest pain, blood-tinged secretions, hypoxemia, and respiratory acidosis. Patients with moderate pulmonary contusions often have a constant, but ineffective cough and cannot clear their secretions

In relation to the structure of the larynx, the cricoid cartilage is which of the following? a) The only complete cartilaginous ring in the larynx b) The largest of the cartilage structures c) Used in vocal cord movement with the thyroid cartilage d) The valve flap of cartilage that covers the opening to the larynx during swallowing

A. The cricoid cartilage is the only complete cartilaginous ring in the larynx (located below the thyroid cartilage). The arytenoid cartilages are used in vocal cord movement with the thyroid cartilage. The thyroid cartilage is the largest of the cartilage structures; part of it forms the Adam's apple. The epiglottis is the valve flap of cartilage that covers the opening to the larynx during swallowing.

A patient comes to the clinic complaining of a possible upper respiratory infection. What should the nurse inspect that would indicate that an upper respiratory infection may be present? a) The nasal mucosa b) The buccal mucosa c) The tracheal mucosa d) The frontal sinuses

A. The nurse inspects the nasal mucosa for abnormal findings such as increased redness, swelling, exudate, and nasal polyps, which may develop in chronic rhinitis. The mucosa of the nasal turbinates may also be swollen (boggy) and pale bluish-gray. The nurse palpates the frontal and maxillary sinuses for tenderness, which suggests inflammation, and then inspects the throat by having the patient open the mouth wide and take a deep breath.

You are caring for a client who has been diagnosed with viral pneumonia. You are making a plan of care for this client. What nursing interventions would you put into the plan of care for a client with pneumonia? a) Encourage increased fluid intake. b) Offer nutritious snacks 2 times a day. c) Place client on bed rest. d) Give antibiotics as ordered

A. The nurse places the client in semi-Fowler's position to aid breathing and increase the amount of air taken with each breath. Increased fluid intake is important to encourage because it helps to loosen secretions and replace fluids lost through fever and increased respiratory rate. The nurse monitors fluid intake and output, skin turgor, vital signs, and serum electrolytes. He or she administers antipyretics as indicated and ordered. Antibiotics are not given for viral pneumonia. The client's activity level is ordered by the physician, not decided by the nurse

A nurse is providing discharge teaching for a client who had a laryngectomy. Which instruction should the nurse include in her teaching? a) "Cover the stoma whenever you shower or bathe." b) "Wear a tight cloth at the stoma to prevent anything from entering it." c) "Swimming is good exercise after this surgery." d) "Keep the humidity in your house low."

A. The nurse should instruct the client to gently cover the stoma with a loose plastic bib, or even a hand, when showering or bathing to prevent water from entering the stoma. The client should cover the stoma with a loose-fitting, not tight, cloth to protect it. The client should keep his house humidified to prevent irritation of the stoma that can occur in low humidity. The client should avoid swimming, because it's possible for water to enter the stoma and then enter the client's lung, causing him to drown without submerging his face.

You are an occupational health nurse in a large ceramic manufacturing company. How would you intervene to prevent occupational lung disease in the employees of the company? a) Fit all employees with protective masks. b) Give workshops on disease prevention. c) Insist on adequate breaks for each employee. d) Provide employees with smoking cessation materials.

A. The primary focus is prevention, with frequent examination of those who work in areas of highly concentrated dust or gases. Laws require work areas to be safe in terms of dust control, ventilation, protective masks, hoods, industrial respirators, and other protection. Workers are encouraged to practice healthy behaviors, such as quitting smoking. Adequate breaks, giving workshops, and providing smoking cessation materials do not prevent occupational lung diseases.

A client comes into the Emergency Department with epistaxis. What intervention should you perform when caring for a client with epistaxis? a) Apply direct continuous pressure. b) Apply a moustache dressing. c) Provide a nasal splint. d) Place the client in a semi-Fowler's position.

A. The severity and location of bleeding determine the treatment of a client with epistaxis. To manage this condition, the nurse should apply direct continuous pressure to the nares for 5 to 10 minutes with the client's head tilted slightly forward. Application of a moustache dressing or a drip pad to absorb drainage, application of a nasal splint, and placement of the client in a semi-Fowler's position are interventions related to the management of a client with a nasal obstruction.

A young adult visited a clinic because he was injured during a softball game. He told the nurse that the ball struck him in his "Adam's apple." To assess the initial impact of injury, the nurse: a) Palpates the thyroid cartilage. b) Inspects the epiglottis. c) Inspects the vocal cords. d) Palpates the cricoid cartilage.

A. The term "Adam's Apple" is used to refer to a lump or protrusion, a laryngeal prominence. It is formed by the angle of the thyroid cartilage surrounding the larynx

Knowing respiratory physiology is important to understand how the disease process can work within that system. Which hollow tube transports air from the laryngeal pharynx to the bronchi? a) Trachea b) Larynx c) Bronchioles d) Pharynx

A. The trachea is a hollow tube composed of smooth muscle and supported by C-shaped cartilage. The trachea transports air from the laryngeal pharynx to the bronchi and lungs. This is a cartilaginous framework between the pharynx and trachea that produces sound. The bronchioles are smaller subdivisions of bronchi within the lungs. The pharynx, or throat, carries air from the nose to the larynx and food from the mouth to the esophagus.

A nurse practitioner diagnosed a patient with an infection in the maxillary sinuses. Select the area that the nurse palpated to make that diagnosis. a) On the cheeks below the eyes b) Above the eyebrows c) Between the eyes and behind the nose d) Behind the ethmoid sinuses

A. To palpate the maxillary sinuses, the nurse should apply gentle pressure in the cheek area below the eyes, adjacent to the nose.

You are caring for a 65-year-old client who has been newly diagnosed with emphysema. The client is confused by the new terms and wants to know what ventilation means. Which of the following can instruct this client? a) Ventilation is breathing air in and out of the lungs. b) Ventilation helps clients who cannot breathe on their own. c) Ventilation provides a blood supply to the lungs. d) Ventilation is when the body changes oxygen into CO2.

A. Ventilation is the actual movement of air in and out of the respiratory tract. Diffusion is the exchange of oxygen and CO2 through the alveolar-capillary membrane. Pulmonary perfusion refers to blood supply to the lungs. A mechanical ventilator assists patients who are unable to breathe on their own.

A new ICU nurse is observed by her preceptor entering a patient's room to suction the tracheostomy after performing the task 15 minutes before. What should the preceptor educate the new nurse to do to ensure that the patient needs to be suctioned? a) Auscultate the lung for adventitious sounds. b) Have the patient inform the nurse of the need to be suctioned. c) Have the patient cough. d) Assess the CO2 level to determine if the patient requires suctioning.

A. When a tracheostomy or endotracheal tube is in place, it is usually necessary to suction the patient's secretions because of the decreased effectiveness of the cough mechanism. Tracheal suctioning is performed when adventitious breath sounds are detected or whenever secretions are obviously present. Unnecessary suctioning can initiate bronchospasm and cause mechanical trauma to the tracheal mucosa.

The client is returning from the operating room following a broncho scopy. Which action, performed by the nursing assistant, would the nurse stop if began prior to nursing assessment? a) The nursing assistant is pouring a glass of water to wet the client's mouth. b) The nursing assistant is assisting the client to a semi-Fowler's position. c) The nursing assistant is assisting the client to the side of the bed to use a urinal. d) The nursing assistant is asking a question requiring a verbal response.

A. When completing a procedure which sends a scope down the throat, the gag reflex is anesthetized to reduce discomfort. Upon returning to the nursing unit, the gag reflex must be assessed before providing any food or fluids to the client. The client may need assistance following the procedure for activity and ambulation but this is not restricted in the post procedure period

The nurse is caring for a patient diagnosed with pneumonia. The nurse will assess the patient for tactile fremitus by completing which of the following? a) Asking the patient to repeat "ninety-nine" as the nurse's hands move down the patient's thorax b) Asking the patient to say "one, two, three" while auscultating the lungs c) Instructing the patient to take a deep breath and hold it while the diaphragm is percussed d) Placing the thumbs along the costal margin of the chest wall and instructing the patient to inhale deeply

A. While the nurse is assessing for tactile fremitus, the patient is asked to repeat "ninety-nine" or "one, two, three," or "eee, eee, eee" as the nurse's hands move down the patient's thorax. The vibrations are detected with the palmar surfaces of the fingers and hands, or the ulnar aspect of the extended hands, on the thorax. The hand or hands are moved in sequence down the thorax. Corresponding areas of the thorax are compared. Asking the patient to say "one, two, three" while auscultating the lungs is not the proper technique for assessing for tactile fremitus. The nurse assesses for anterior respiratory excursion by placing the thumbs along the costal margin of the chest wall and instructing the patient to inhale deeply. The nurse assesses for diaphragmatic excursion by instructing the patient to take a deep breath and hold it while the diaphragm is percussed

The nurse is instructing the patient with asthma in the use of a newly prescribed leukotriene receptor antagonist. What should the nurse be sure to include in the education? a) The patient should take the medication an hour before meals or 2 hours after a meal. b) The patient should take the medication with meals since it may cause nausea. c) The patient should take the medication with a small amount of liquid. d) The patient should take the medication separately without other medications.

A. he nurse should instruct the patient to take the leukotriene receptor antagonist at least 1 hour before meals or 2 hours after meals.

The nurse is working on a busy respiratory unit. In caring for a variety of clients, the nurse must be knowledgeable of diagnostic studies. With which diagnostic studies would the nurse screen the client for an allergy to iodine? Select all that apply. a) Chest x-ray b) Lung scan c) Pulmonary angiography d) Fluoroscopy e) Pulmonary functions test f) Bronchoscopy

B, C, D. The nurse must be well educated in screening clients before diagnostic procedures which include contrast medium for an allergy to iodine. A lung scan, fluoroscopy and pulmonary angiography all require contrast medium

Which of the following are indicators that a client is ready to be weaned from a ventilator? Select all that apply. a) FiO2 45% b) Tidal volume of 8.5 mL/kg c) PaO2 of 64 mm Hg d) Vital capacity of 13 mL/kg e) Rapid/shallow breathing index of 112 breaths/min

B, C, D. Weaning criteria for clients are as follows: Vital capacity 10 to 15 mL/kg; Maximum inspiratory pressure at least -20 cm H2; Tidal volume: 7 to 9 mL/kg; Minute ventilation: 6 L/min; Rapid/shallow breathing index below 100 breaths/min; PaO2 > 60 mm Hg; FiO2 < 40%

The nurse is educating the patient in the use of a mini-nebulizer. What should the nurse encourage the patient to do? (Select all that apply.) a) Take rapid, deep breaths. b) Cough frequently. c) Frequently evaluate progress. d) Hold the breath at the end of inspiration for a few seconds. e) Prolong the expiratory phase after using the nebulizer.

B, C, D. The nurse instructs the patient to breathe through the mouth, taking slow, deep breaths, and then to hold the breath for a few seconds at the end of inspiration to increase intrapleural pressure and reopen collapsed alveoli, thereby increasing functional residual capacity. The nurse encourages the patient to cough and to monitor the effectiveness of the therapy. The nurse instructs the patient and family about the purpose of the treatment, equipment setup, medication additive, and proper cleaning and storage of the equipment

The nurse is caring for a geriatric client brought to the emergency department after being found by her children feeling poorly with an elevated temperature. Laboratory tests confirm influenza type A, a respiratory virus. Which medical treatment would the nurse anticipate in the discharge instructions? Select all that apply. a) Antibiotics b) Saline gargles c) Increased fluids d) Antiemetics e) Antitussives f) Rest

B, C, E, F. Influenza type A is the most common cause of the flu initiated by a respiratory virus. Common discharge instructions include rest, increased fluids to thin respiratory secretions, saline gargles to help prevent a throat infection such a strep throat, and antitussives if the client is coughing. Antibiotics are not used with a virus unless a bacterial infection subsequently develops. Antiemetics are used for nausea and vomiting not commonly associated with a common respiratory virus

The nurse is caring for a client with a new tracheostomy. Which of the following nursing diagnoses are priorities? Select all that apply. a) Risk for Infection related to operative incision and tracheostomy tube placement b) Ineffective Airway Clearance related to increased secretions c) Knowledge Deficit related to care of the tracheostomy tube and surrounding site d) Impaired Gas Exchange related to shallow breathing and anxiousness

B, D. The client with a new tracheostomy tube has increased secretions, which may become dried and occlude the airway or plug the airway requiring frequent suctioning. Impaired Gas Exchange is an equally important diagnosis. These are related to airway and breathing and are priorities.

A 72-year-old male client finished a course of antibiotics for laryngitis but continues to experience persistent hoarseness. If laryngeal cancer is suspected, the nurse would be most likely to hear which of the following complaints from the client? a) Weight loss b) A feeling of swelling at the back of the throat c) Discomfort when drinking cold liquids d) Headaches in the morning

B. After an initial hoarseness lasting longer than a month, clients with laryngeal cancer will feel a sensation of swelling or a lump in the throat or in the neck. Weight loss often occurs later in the progression of laryngeal cancer due to reduced calorie intake as a result of impaired swallowing and pain. Clients with laryngeal cancer may complain of burning in the throat when swallowing hot or citrus liquids. Clients with obstructive sleep apnea may experience a morning headache

A nurse is concerned that a client may develop postoperative atelectasis. Which nursing diagnosis would be most appropriate if this complication occurs? a) Impaired spontaneous ventilation b) Impaired gas exchange c) Ineffective airway clearance d) Decreased cardiac output

B. Airflow is decreased with atelectasis, which is a bronchial obstruction from collapsed lung tissue. If there is an obstruction, there is limited or no gas exchange in this area. Impaired gas exchange is thus the most likely nursing diagnosis with atelectasis.

A client is being seen in the emergency department for exacerbation of chronic obstructive pulmonary disease (COPD). The first action of the nurse is to administer which of the following prescribed treatments? a) Vancomycin 1 gram intravenously over 1 hour b) Oxygen through nasal cannula at 2 L/minute c) Intravenous methylprednisolone (Solu-Medrol) 120 mg d) Ipratropium bromide (Alupent) by metered-dose inhaler

B. All options listed are treatments that may be used for a client with an exacerbation of COPD. The first line of treatment is oxygen therapy.

A client with chronic obstructive pulmonary disease (COPD) expresses a desire to quit smoking. The first appropriate response from the nurse is: a) "I can refer you to the American Lung Association." b) "Have you tried to quit smoking before?" c) "Many options are available for you." d) "Nicotine patches would be appropriate for you."

B. All the options are appropriate statements; however, the nurse needs to assess the client's statement further. Assessment data include information about previous attempts to quit smoking.

The nurse suctions a patient through the endotracheal tube for 20 seconds and observes dysrhythmias on the monitor. What does the nurse determine is occurring with the patient? a) The patient is in a hypermetabolic state. b) The patient is hypoxic from suctioning. c) The patient is having a myocardial infarction. d) The patient is having a stress reaction.

B. Apply suction while withdrawing and gently rotating the catheter 360 degrees (no longer than 10-15 seconds). Prolonged suctioning may result in hypoxia and dysrhythmias, leading to cardiac arrest.

As status asthmaticus worsens, the nurse would expect which acid-base imbalance? a) Metabolic alkalosis b) Respiratory acidosis c) Respiratory alkalosis d) Metabolic acidosis

B. As status asthmaticus worsens, the PaCO increases and the pH decreases, reflecting respiratory acidosis.

A patient is diagnosed with mild obstructive sleep apnea after having a sleep study performed. What treatment modality will be the most effective for this patient? a) Surgery to remove the tonsils and adenoids b) Continuous positive airway pressure c) Medications to assist the patient with sleep at night d) Bi-level positive airway pressure

B. Continuous positive airway pressure (CPAP) provides positive pressure to the airways throughout the respiratory cycle. Although it can be used as an adjunct to mechanical ventilation with a cuffed endotracheal tube or tracheostomy tube to open the alveoli, it is also used with a leak-proof mask to keep alveoli open, thereby preventing respiratory failure. CPAP is the most effective treatment for obstructive sleep apnea because the positive pressure acts as a splint, keeping the upper airway and trachea open during sleep. To use CPAP, the patient must be breathing independently

The nurse is providing discharge instructions to a patient with pulmonary sarcoidosis. The nurse concludes that the patient understands the information if the patient correctly states which of the following early signs of exacerbation? a) Fever b) Shortness of breath c) Headache d) Weight loss

B. Early signs and symptoms of pulmonary sarcoidosis may include dyspnea, cough, hemoptysis, and congestion. Generalized symptoms include anorexia, fatigue, and weight loss

For a client with an endotracheal (ET) tube, which nursing action is the most important? a) Turning the client from side to side every 2 hours b) Auscultating the lungs for bilateral breath sounds c) Providing frequent oral hygiene d) Monitoring serial blood gas values every 4 hours

B. For the client with an ET tube, the most important nursing action is auscultating the lungs regularly for bilateral breath sounds to ensure proper tube placement and effective oxygen delivery. Although turning the client from side to side every 2 hours, monitoring serial blood gas values every 4 hours, and providing frequent oral hygiene are appropriate actions for this client, they're secondary to ensuring adequate oxygenation.

The nurse is caring for a patient who is to undergo a thoracentesis. In preparation for the procedure, the nurse will position the patient in which of the following positions? a) Lateral recumbent b) Sitting on the edge of the bed c) Prone d) Supine

B. If possible, it is best to place the patient upright or sitting on the edge of the bed with the feet supported and arms and head on a padded over-the-bed table. Other positions in which the patient could be placed include straddling a chair with arms and head resting on the back of the chair, or lying on the unaffected side with the head of the bed elevated 30 to 45 degrees if unable to assume a sitting position.

The nurse should be alert for a complication of bronchiectasis that results from a combination of retained secretions and obstruction and that leads to the collapse of alveoli. What complication should the nurse monitor for? a) Pleurisy b) Atelectasis c) Pneumonia d) Emphysema

B. In bronchiectasis, the retention of secretions and subsequent obstruction ultimately cause the alveoli distal to the obstruction to collapse (atelectasis).

Another term for clergyman's sore throat is a) aphonia. b) chronic granular pharyngitis. c) atrophic pharyngitis. d) hypertrophic pharyngitis.

B. In clergyman's sore throat, the pharynx is characterized by numerous swollen lymph follicles. Aphonia refers to the inability to use one's voice. Atrophic pharyngitis is characterized by a membrane that is thin, white, glistening, and at times wrinkled. Hypertrophic pharyngitis is characterized by general thickening and congestion of the pharyngeal mucous membrane

A client with acute viral rhinosinusitis is being seen in a clinic. The nurse is providing discharge instructions and includes the following information: a) Dehumidify the air in the home. b) Avoid air travel. c) Severe pain when palpating the sinuses is normal. d) Make sure to follow antibiotic therapy.

B. Information that the nurse should include for a client with acute viral rhinosinusitis is to avoid air travel. Other nursing interventions include referring the client to a physician if severe pain occurs when palpating the sinuses and humidifying the air in the home to promote drainage. Antibiotic therapy is not indicated for a viral infection

Which of the following terms will the nurse use to document the inability of a patient to breathe easily unless positioned upright? a) Hemoptysis b) Orthopnea c) Hypoxemia d) Dyspnea

B. Orthopnea is the term used to describe a patient's inability to breathe easily except in an upright position. Orthopnea may be found in patients with heart disease and, occasionally, in patients with COPD. Patients with orthopnea are placed in a high Fowler's position to facilitate breathing. Dyspnea refers to labored breathing or shortness of breath. Hemoptysis refers to expectoration of blood from the respiratory tract. Hypoxemia refers to low oxygen levels in the blood.

Nursing instruction during postural drainage should include telling the patient to: a) Remain in each position for 30 to 45 minutes for best results. b) Change positions frequently and cough up secretions. c) Lie supine to rest the lungs. d) Sit upright to promote ventilation

B. Patients who lie supine will have secretions accumulate in the posterior lung sections, whereas upright patients will pool secretions in their lower lobes. By changing positions (see Fig. 10-2 in the text), secretions can drain from the affected bronchioles into the bronchi and trachea and then be removed by coughing or suctioning

The nurse is assisting a client with postural drainage. Which of the following demonstrates correct implementation of this technique? a) Administer bronchodilators and mucolytic agents following the sequence. b) Instruct the client to remain in each position of the postural drainage sequence for 10 to 15 minutes. c) Perform this measure with the client once a day. d) Use aerosol sprays to deodorize the client's environment after postural drainage.

B. Postural drainage is usually performed two to four times daily, before meals (to prevent nausea, vomiting, and aspiration) and at bedtime. Prescribed bronchodilators, water, or saline may be nebulized and inhaled before postural drainage to dilate the bronchioles, reduce bronchospasm, decrease the thickness of mucus and sputum, and combat edema of the bronchial walls. The nurse instructs the client to remain in each position for 10 to 15 minutes and to breathe in slowly through the nose and out slowly through pursed lips to help keep the airways open so that secretions can drain while in each position. If the sputum is foul-smelling, it is important to perform postural drainage in a room away from other patients or family members. (Deodorizers may be used to counteract the odor. Because aerosol sprays can cause bronchospasm and irritation, they should be used sparingly and with caution.)

A client has acute bacterial rhinosinusitis for several weeks despite treatment. The nurse observes for a possible complication of the infection by assessing for a) Nausea b) Nuchal rigidity c) Hypertension d) Mild headache

B. Potential complications of acute bacterial rhinosinusitis are nuchal rigidity and severe headache. Hypertension may be a result of over-the-counter decongestant medications. Nausea may be a result of nasal corticosteroids

During assessment of a patient with OSA, the nurse documents which of the following characteristic signs that occurs because of repetitive apneic events? a) Increased smooth muscle contractility b) Hypercapnia c) Systemic hypotension d) Pulmonary hypotension

B. Repetitive apneic events result in hypoxia and hypercapnia, which triggers a sympathetic response (increased heart rate and decreased tone and contractility of smooth muscle)

A nurse is reviewing arterial blood gas results on an assigned client. The pH is 7.32 with PCO2 of 49 mm Hg and a HCO3−of 28 mEq/L. The nurse reports to the physician which finding? a) Respiratory alkalosis b) Respiratory acidosis c) Metabolic alkalosis d) Metabolic acidosis

B. Respiratory acidosis would be reported to the physician citing the lab values. Analysis of the blood gases reveals that the client is acidotic with a pH under 7.35. Also noted is the PCO2above the normal range of 30 to 40 mm Hg. The HCO3− is slightly elevated because the normal level is 22 to 26 mEq/L

A client with chronic bronchitis is admitted to the health facility. Auscultation of the lungs reveals low-pitched, rumbling sounds. Which of the following describes these sounds? a) Rales b) Rhonchi c) Venous hum d) Bronchovesicular

B. Rhonchi are deep, low-pitched, rumbling sounds heard usually on expiration. The etiology of rhonchi is associated with chronic bronchitis.

The nurse is caring for a patient with an endotracheal tube (ET). Which of the following nursing interventions is contraindicated? a) Checking the cuff pressure every 6 to 8 hours b) Deflating the cuff routinely c) Deflating the cuff prior to tube removal d) Ensuring that humidified oxygen is always introduced through the tube

B. Routine cuff deflation is not recommended because of the increased risk for aspiration and hypoxia. The cuff is deflated before the ET is removed. Cuff pressures should be checked every 6 to 8 hours. Humidified oxygen should always be introduced through the tube

A client with suspected severe acute respiratory syndrome (SARS) comes to the emergency department. Which physician order should the nurse implement first? a) Obtain a nasopharyngeal specimen for reverse-transcription polymerase chain reaction testing. b) Institute isolation precautions. c) Obtain a sputum specimen for enzyme immunoassay testing. d) Begin an I.V. infusion of dextrose 5% in half-normal saline solution at 100 ml/hour.

B. SARS, a highly contagious viral respiratory illness, is spread by close person-to-person contact. Contained in airborne respiratory droplets, the virus is easily transmitted by touching surfaces and objects contaminated with infectious droplets. The nurse should give top priority to instituting infection-control measures to prevent the spread of infection to emergency department staff and clients. After isolation measures are carried out, the nurse can begin an I.V. infusion of dextrose 5% in half-normal saline and obtain nasopharyngeal and sputum specimens

The pathophysiology of emphysema is directly related to airway obstruction. The end result of deterioration is respiratory acidosis from airway obstruction. Knowing this, the nursing student would choose which of the following as the highest priority action? a) Educate the importance of pursed lip breathing. b) Assess vital signs every 2 hours including O2 saturations and ABG results. c) Refer to respiratory therapy if breathing becomes labored. d) Apply oxygen as ordered via nasal cannula.

B. The acuity of the onset and severity of respiratory failure depend on baseline pulmonary function, pulse oximetry or arterial blood gas values, comorbid conditions, and the severity of other complications of COPD

The most diagnostic clinical symptom of pleurisy is: a) Dyspnea and coughing. b) Stabbing pain during respiratory movements. c) Dullness or flatness on percussion over areas of collected fluid. d) Fever and chills.

B. The key characteristic of pleuritic pain is its relationship to respiratory movement: taking a deep breath, coughing, or sneezing worsens the pain. Pleuritic pain is limited in distribution rather than diffuse; it usually occurs only on one side. The pain may become minimal or absent when the breath is held; leading to rapid shallow breathing. It may be localized or radiate to the shoulder or abdomen. Later, as pleural fluid accumulates, the pain decreases

A 6-month-old male client and his elder brother, a 3-year-old male, are being seen in the pediatric clinic for their third middle ear infection of the winter. The mother reports they develop an upper respiratory infection and an ear infection seems quick to follow. What contributes to this event? a) Oropharynx b) Eustachian tubes c) Genetics d) Epiglottis

B. The nasopharynx contains the adenoids and openings of the eustachian tubes. The eustachian tubes connect the pharynx to the middle ear and are the means by which upper respiratory infections spread to the middle ear. The client's infection is not caused by genetics. The oropharynx contains the tongue. The epiglottis closes during swallowing and relaxes during respiration

A 62-year-old male client with a history of chronic laryngitis arrives at the clinic complaining of a hoarseness "he can't shake." The nurse is aware that this client may be at risk for which of the following conditions? a) Coryza b) Laryngeal cancer c) Adenoiditis d) Peritonsillar abscess

B. The nurse knows that laryngeal cancer is most common in people 60 to 70 years of age, with men affected more frequently than are women. The client's history of chronic laryngitis may also predispose him to the development of laryngeal cancer. Sore throat, difficulty or pain on swallowing, fever, and malaise are the most common symptoms of adenoiditis. Enlarged adenoids may produce nasal obstruction, noisy breathing, snoring, and a nasal quality to the voice. This is another term for the common cold. Symptoms include sneezing, sore throat, and nasal congestions. Clients with a peritonsillar abscess experience difficulty and pain with swallowing, fever, malaise, ear pain, and difficulty talking

A 53-year-old male is a regular client in the respiratory group where you practice nursing. As with all adults, millions of alveoli form most of the pulmonary mass. The squamous epithelial cells lining each alveolus consist of different types of cells. Which type of the alveoli cells produce surfactant? a) Type IV cells. b) Type II cells c) Type III cells. d) Type I cells

B. Type II cells—produce surfactant, a phospholipid that alters the surface tension of alveoli, preventing their collapse during expiration and limiting their expansion during inspiration. Type I cells line most alveolar surfaces. The epithelium of the alveoli does not contain Type IV cells. Type III cells destroy foreign material, such as bacteria

A nurse is weaning a client from mechanical ventilation. Which assessment finding indicates the weaning process should be stopped? a) Oxygen saturation of 93% b) Runs of ventricular tachycardia c) Respiratory rate of 16 breaths/minute d) Blood pressure increase from 120/74 mm Hg to 134/80 mm Hg

B. Ventricular tachycardia indicates that the client isn't tolerating the weaning process. The weaning process should be stopped before lethal ventricular arrhythmias occur. A respiratory rate of 16 breaths/minute and an oxygen saturation of 93% are normal findings. Although the client's blood pressure has increased, it hasn't increased more than 20% over baseline, which would indicate that the client isn't tolerating the weaning process

A client experiencing an asthmatic attack is prescribed methylprednisolone (Solu-Medrol) intravenously. The nurse: a) Aspirates for blood return before injecting the medication b) Informs the client to limit fluid intake due to fluid retention c) Assesses fasting blood glucose levels d) Encourages the client to decrease caloric intake due to increased appetite

C. Adverse effects of methylprednisolone include abnormalities in glucose metabolism. The nurse monitors blood glucose levels. Methylprednisolone also increases the client's appetite and fluid retention, but the client will not decrease caloric or fluid intake as a result of these adverse effects. It is not necessary to aspirate for blood return prior to injecting the medication, because doing so would not support the intravenous line in the vein

You are caring for a client who is 42-years-old and status post adenoidectomy. You find the client in respiratory distress when you enter their room. You ask another nurse to call the physician and bring an endotracheal tube into the room. What do you suspect? a) Post operative bleeding b) Plugged tracheostomy tube c) Edema of the upper airway d) Infection

C. An endotracheal tube is inserted through the mouth or nose into the trachea to provide a patent airway for clients who cannot maintain an adequate airway on their own. The scenario does not indicate infection, post operative bleeding, or a plugged tracheostomy tube

A nursing student understands the importance of the psychosocial aspects of disease processes. When working with a patient with COPD, the student would rank which of the following nursing diagnoses as the MOST important when analyzing the psychosocial effects? a) High risk for ineffective therapeutic regimen management related to lack of knowledge b) Disturbed sleep pattern related to cough c) Ineffective coping related to anxiety d) Activity intolerance related to fatigue

C. Any factor that interferes with normal breathing quite naturally induces anxiety, depression, and changes in behavior. Constant shortness of breath and fatigue may make the patient irritable and apprehensive to the point of panic. Although the other choices are correct, the most important psychosocial nursing diagnosis for a patient with COPD is ineffective coping related to a high level of anxiety.

A 65-year-old client who works construction, and has been demolishing an older building,is diagnosed with pneumoconiosis. The nurse is aware that his lung inflammation is most likely caused by exposure to which of the following? a) Silica b) Pollen c) Asbestos d) Coal dust

C. Asbestosis is caused by inhalation of asbestos dust, which is frequently encountered during construction work, particularly when working with older buildings. Laws restrict asbestos use, but old materials still contain asbestos. Inhalation of silica may cause silicosis, which results from inhalation of silica dust and is seen in workers involved with mining, quarrying, stone-cutting, and tunnel building. Inhalation of coal dust and other dusts may cause black lung disease. Pollen may cause an allergic reaction, but is unlikely to cause pneumoconiosis

Which ventilator mode provides full ventilatory support by delivering a present tidal volume and respiratory rate? a) IMV b) Pressure support c) Assist-control d) SIMV

C. Assist-control ventilation provides full ventilator support by delivering a preset tidal volume and respiratory rate. Intermittent mandatory ventilation (IMV) provides a combination of mechanically assisted breaths and spontaneous breaths. SIMV delivers a preset tidal volume and number of breaths per minute. Between ventilator-delivered breaths, the patient can breathe spontaneously with no assistance from the ventilator for those extra breaths.

A client is admitted to the health care facility with active tuberculosis (TB). The nurse should include which intervention in the care plan? a) Wearing a gown and gloves when providing direct care b) Instructing the client to wear a mask at all times c) Putting on an individually fitted mask when entering the client's room d) Keeping the door to the client's room open to observe the client

C. Because TB is transmitted by droplet nuclei from the respiratory tract, the nurse should put on a mask when entering the client's room. Occupation Safety and Health Administration standards require an individually fitted mask. Having the client wear a mask at all times would hinder sputum expectoration and respirations would make the mask moist. A nurse who doesn't anticipate contact with the client's blood or body fluids need not wear a gown or gloves when providing direct care. A client with TB should be in a room with laminar airflow, and the room's door should be shut at all times.

A junior-level nursing class has just finished learning about the management of clients with chronic pulmonary diseases. They learned that a new definition of COPD leaves only one disorder within its classification. Which of the following is that disorder? a) Asthma b) Bronchiectasis c) Emphysema d) Cystic fibrosis

C. COPD may include diseases that cause airflow obstruction (eg, emphysema, chronic bronchitis) or any combination of these disorders. Other diseases such as cystic fibrosis, bronchiectasis, and asthma that were previously classified as types of COPD are now classified as chronic pulmonary disorders. Asthma is now considered a distinct, separate disorder and is classified as an abnormal airway condition characterized primarily by reversible inflammation

A thoracentesis is performed to obtain a sample of pleural fluid or a biopsy specimen from the pleural wall for diagnostic purposes. What does serous fluid indicate? A. Trauma B. Infection C. Cancer D.

C. Cancer. A thoracentesis may be performed to obtain a sample of pleural fluid or to biopsy a specimen from the pleural wall for diagnostic purposes. Serous fluid may be associated with cancer, inflammatory conditions, or heart failure. Blood fluid typically suggests trauma. Purulent fluid is diagnostic for infection. Complications that may follow a thoracentesis include pneumothorax and subcutaneous emphysema.

A client suffers acute respiratory distress syndrome as a consequence of shock. The client's condition deteriorates rapidly, and endotracheal (ET) intubation and mechanical ventilation are initiated. When the high-pressure alarm on the mechanical ventilator sounds, the nurse starts to check for the cause. Which condition triggers the high-pressure alarm? a) An ET cuff leak b) A change in the oxygen concentration without resetting the oxygen level alarm c) Kinking of the ventilator tubing d) A disconnected ventilator tube

C. Conditions that trigger the high-pressure alarm include kinking of the ventilator tubing, bronchospasm, pulmonary embolus, mucus plugging, water in the tube, and coughing or biting on the ET tube. The alarm may also be triggered when the client's breathing is out of rhythm with the ventilator. A disconnected ventilator tube or an ET cuff leak would trigger the low-pressure alarm. Changing the oxygen concentration without resetting the oxygen level alarm would trigger the oxygen alarm, not the high-pressure alarm.

Which of the following interventions regarding nutrition is implemented for patients who have undergone laryngectomy? a) Season food to suit an increased sense of taste and smell b) Recommend the long-term use of zinc lozenges c) Use enteral feedings after the procedure d) Offer plenty of thin liquids when intake resumes

C. Enteral feedings are used 10 to 14 days after a laryngectomy to avoid irritation to the sutures and reduce the risk of aspiration. When oral intake resumes, the nurse offers small amounts of thick liquids. Following a laryngectomy, the patient may experience anorexia related to a diminished sense of taste and smell. Excess zinc can impair the immune system and lower the levels of high-density lipoproteins ("good" cholesterol). Therefore, long-term or ongoing use of zinc lozenges to prevent a cold is not recommended

What client would be most in need of an endotracheal tube? a) Ambulatory clients b) Older adult clients c) Comatose clients d) A client status post tonsillectomy

C. Examples include those with respiratory difficulty, comatose clients, those undergoing general anesthesia, and clients with extensive edema of upper airway passages

You are an occupational nurse completing routine assessments on the employees where you work. What might be revealed by a chest radiograph for a client with occupational lung diseases? a) Lung contusion b) Hemorrhage c) Fibrotic changes in lungs d) Damage to surrounding tissues

C. For a client with occupational lung diseases, a chest radiograph may reveal fibrotic changes in the lungs. Hemorrhage, lung contusion, and damage to surrounding tissues are possibly caused by trauma due to chest injuries

A nurse takes the initial history of a patient who is being examined for cancer of the larynx. Select the sign that is considered an early clinical indicator. a) Cervical lymph adenopathy b) Dysphasia c) Hoarseness of more than 2 week's duration d) Persistent ulceration

C. Hoarseness of more than 2 weeks' duration occurs in the patient with cancer in the glottic area, because the tumor impedes the action of the vocal cords during speech. The voice may sound harsh, raspy, and lower in pitch. Later symptoms include dysphasia, dyspnea (difficulty breathing), unilateral nasal obstruction or discharge, persistent hoarseness, persistent ulceration, and foul breath. Cervical lymph adenopathy, unintentional weight loss, a general debilitated state, and pain radiating to the ear may occur with metastasis

As part of a primary cancer prevention program, an oncology nurse answers questions from the public at a health fair. When someone asks about laryngeal cancer, the nurse should explain that: a) laryngeal cancer occurs primarily in women. b) adenocarcinoma accounts for most cases of laryngeal cancer. c) laryngeal cancer is one of the most preventable types of cancer. d) inhaling polluted air isn't a risk factor for laryngeal cancer.

C. Laryngeal cancer is one of the most preventable types of cancer; it can be prevented by abstaining from excessive drinking and smoking. Inhaling noxious fumes, such as in polluted air, is a risk factor for laryngeal cancer. Roughly 80% of laryngeal cancer cases occur in men. Squamous cell carcinoma accounts for most cases of laryngeal cancer.

A student nurse is working with a client who is diagnosed with head trauma. The nurse has documented Cheyne-Stokes respirations. The student would expect to see which of the following? a) Irregular breathing at 14 to 18 breaths per minute b) Period of cessation of breathing c) Regular breathing where the rate and depth increase, then decrease d) Periods of normal breathing followed by periods of apnea

C. Observing the rate and depth of respiration is an important aspect of the nursing assessment. Certain patterns of breathing are characteristic of specific disease states or conditions. Head trauma can cause damage to the respiratory center in the brain, thereby altering the rate and depth of respirations. Cheyne-Stokes breathing is characterized by a regular cycle in which the rate and depth of breathing increase, then decrease until apnea occurs

A patient describes his chest pain as knife-like on inspiration. Which of the following is the most likely diagnosis? a) A lung infection b) Bronchogenic carcinoma c) Pleurisy d) Bacterial pneumonia

C. Pleuritic pain from irritation of the parietal pleura is sharp and seems to "catch" on inspiration. Some patients describe the pain as being "stabbed by a knife." Chest pain associated with the other conditions may be dull, aching, and persistent.

The nurse is caring for a client diagnosed with asthma. While performing the shift assessment, the nurse auscultates breath sounds including sibilant wheezes, which are continuous musical sounds. What characteristics describe sibilant wheezes? a) They occur when the pleural surfaces are inflamed. b) They are heard in clients with decreased secretions. c) They can be heard during inspiration and expiration. d) They result from air passing through widened air passages.

C. Sibilant or hissing or whistling wheezes are continuous musical sounds that can be heard during inspiration and expiration. They result from air passing through narrowed or partially obstructed air passages and are heard in clients with increased secretions. The crackling or grating sounds heard during inspiration or expiration are friction rubs. They occur when the pleural surfaces are inflamed

Upon palpation of the sinus area, what would the nurse identify as a normal finding? a) Tenderness during palpation b) Pain sensation behind the eyes c) No sensation during palpation d) Light not going through the sinus cavity

C. Sinus assessment involves using the thumbs to apply gentle pressure in an upward fashion at the sinuses. Tenderness suggests inflammation. The sinuses can be inspected by transillumination, where a light is passed through the sinuses. If the light fails to penetrate, the cavity contains fluid

In which stage of COPD is the forced expiratory volume (FEV) less than 30%? a) 0 b) II c) III d) I

C. Stage III patients demonstrate an FEV less than 30% with respiratory failure or clinical signs of right heart failure. Stage II patients demonstrate an FEV of 30% to 80%. Stage I is mild COPD with an FEV less than 70%. Stage 0 is characterized by normal spirometry

The goal for oxygen therapy in COPD is to support tissue oxygenation, decrease the work of the cardiopulmonary system, and maintain the resting partial arterial pressure of oxygen (PaO2) of at least ______ mm Hg and an arterial oxygen saturation (SaO2) of at least ___%. a) 58 mm Hg; 88% b) 56 mm Hg; 86% c) 60 mm Hg; 90% d) 54 mm Hg; 84%

C. The goal is a PaO2 of at least 60 mm Hg and an SaO2 of 90%.

The nurse is providing discharge instructions to a patient following nasal surgery who has nasal packing. Which of the following discharge instructions would be most appropriate for the patient? a) Decrease the amount of daily fluids. b) Administer normal saline nasal drops as ordered. c) Avoid sports activities for 6 weeks. d) Take aspirin for nasal discomfort.

C. The nurse instructs the patient to avoid sports activities for 6 weeks. There is no indication for the patient to refrain from taking oral fluids. Mouth rinses help to moisten the mucous membranes and to reduce the odor and taste of dried blood in the oropharynx and nasopharynx. The patient should take analgesic agents, such as acetaminophen or NSAIDs, (i.e., ibuprofen or naproxen) to decrease nasal discomfort, not aspirin. The patient does not need to use nasal drops when nasal packing is in place.

A nurse prepares to perform postural drainage. How should the nurse ascertain the best position to facilitate clearing the lungs? a) Inspection b) Arterial blood gas (ABG) levels c) Auscultation d) Chest X-ray

C. The nurse should assess breath sounds before doing postural drainage to determine the areas that need draining. Inspection, chest X-rays, and ABG levels are all assessment parameters that give good information about respiratory function but aren't necessary to determine lung areas requiring postural drainage

You are doing preoperative teaching with a client scheduled for laryngeal surgery. What should you teach this client to help prevent atelectasis? a) Caution against frequent coughing. b) Monitor for signs of dysphagia. c) Encourage deep breathing every 2 hours. d) Provide meticulous mouth care every 4 hours.

C. The nurse should encourage a client undergoing laryngeal surgery to practice deep breathing and coughing every 2 hours while the client is awake. These measures prevent atelectasis and promote effective gas exchange. Monitoring for signs of dysphagia and providing meticulous mouth care every 4 hours are the interventions related to the client's caloric intake.

The nursing instructor is teaching a pre-nursing pathophysiology class. The class is covering the respiratory system. The instructor explains that the respiratory system is comprised of both the upper and lower respiratory system. The nose is part of the upper respiratory system. The instructor continues to explain that the nasal cavities have a vascular and ciliated mucous lining. What is the purpose of the vascular and ciliated mucous lining of the nasal cavities? a) Move mucus to the back of the throat b) Cool and dry expired air c) Warm and humidify inspired air d) Moisten and filter expired air

C. The vascular and ciliated mucous lining of the nasal cavities warms and humidifies inspired air. It is the function of the cilia alone to move mucus in the nasal cavities and filter the inspired air

The body of a critically ill client may use which of the following homeostatic mechanisms to maintain normal pH? a) The lungs increase respiratory volume. b) The lungs retain more CO2 to lower the pH. c) The lungs eliminate carbonic acid by blowing off more CO2. d) The kidneys retain more HCO3 to raise the pH.

C. To maintain normal pH in critically ill clients, the lungs eliminate carbonic acid by blowing off more CO2. To maintain normal pH in critically ill clients, the lungs conserve CO2 by slowing respiratory volume. This is the way the body would compensate during an acid-base imbalance in cases of metabolic alkalosis. This is the way the body would compensate during an acid-base imbalance in cases of metabolic acidosis

A patient comes to the clinic with complaints of a sore throat and is diagnosed with acute pharyngitis. What does the nurse understand is the cause of acute pharyngitis? a) Gram-negative Klebsiella b) Pseudomonas aeruginosa c) Group A, beta-hemolytic streptococci d) Staphylococcus aureus

C. Viral infection causes most cases of acute pharyngitis. Responsible viruses include the adenovirus, influenza virus, Epstein-Barr virus, and herpes simplex virus. Bacterial infection accounts for the remainder of cases. Ten percent of adults with pharyngitis have group A beta-hemolytic streptococcus (GABHS), which is commonly referred to as group A streptococcus (GAS) or streptococcal pharyngitis

The nurse is auscultating the patient's lung sounds to determine the presence of pulmonary edema. What adventitious lung sounds are significant for pulmonary edema? a) Low-pitched rhonchi during expiration b) Sibilant wheezes c) Crackles in the lung bases d) Pleural friction rub

C. When clinically significant atelectasis develops, it is generally characterized by increased work of breathing and hypoxemia. Decreased breath sounds and crackles are heard over the affected area.

The nurse working in the radiology clinic is assisting with a pulmonary angiography. The nurse knows that when monitoring clients after a pulmonary angiography, what should the physician be notified about? a) Excessive capillary refill b) Flushed feeling in the client c) Absent distal pulses d) Raised temperature in the affected limb

C. When monitoring clients after a pulmonary angiography, nurses must notify the physician about diminished or absent distal pulses, cool skin temperature in the affected limb, and poor capillary refill. When the contrast medium is infused, the client will sense a warm, flushed feeling.

Which type of ventilator has a pre-sent volume of air to be delivered with each inspiration? a) Pressure cycled b) Negative pressure c) Volume cycled d) Time cycled

C. With volume-cycled ventilation, the volume of air to be delivered with each inspiration is present. Negative pressure ventilators exert a negative pressure on the external chest. Time-cycled ventilators terminate or control inspiration after a preset time. When the pressure-cycled ventilator cycles on, it delivers a flow of air (inspiration) until it reaches a present pressure, and then cycles off, and expiration occurs passively

Hyperbaric oxygen therapy increases the blood's capacity to carry and deliver oxygen to compromised tissues. This therapy may be used for a client with: a) hyperthermia. b) a malignant tumor. c) pneumonia. d) a compromised skin graft.

D. A client with a compromised skin graft could benefit from hyperbaric oxygen therapy because increasing oxygenation at the wound site promotes wound healing. Hyperbaric oxygen therapy isn't indicated for malignant tumors, pneumonia, or hyperthermia.

A client is being discharged from an outpatient surgery center following a tonsillectomy. The nurse gives the following instructions: a) "You are allowed to have hot tea or coffee." b) "Gargle vigorously to clean your throat." c) "You may have a sore throat for 1 week." d) "Gargle with a warm salt solution."

D. A warm saline solution will help with removal of thick mucus and halitosis. It will be a gentle gargle, because a vigorous gargle may cause bleeding. A sore throat may be present for 3 to 5 days. Hot foods should be avoided

A client reports nasal congestion, sneezing, sore throat, and coughing up of yellow mucus. The nurse assesses the client's temperature as 100.2°F. The client states this is the third episode this season. The highest priority nursing diagnosis is a) Deficient knowledge related to prevention of upper respiratory infections b) Deficient fluid volume related to increased fluid needs c) Acute pain related to upper airway irritation d) Ineffective airway clearance related to excess mucus production

D. All the listed nursing diagnoses are appropriate for this client. Following Maslow's hierarchy of needs, physiological needs are addressed first and, within physiological needs, airway, breathing, and circulation are the most immediate. Thus, ineffective airway clearance is the priority nursing diagnosis

Before weaning a client from a ventilator, which assessment parameter is the most important for the nurse to obtain? a) Electrocardiogram (ECG) results b) Prior outcomes of weaning c) Fluid intake for the past 24 hours d) Baseline arterial blood gas (ABG) levels

D. Before weaning the client from mechanical ventilation, it's most important to have baseline ABG levels. During the weaning process, ABG levels will be checked to assess how the client is tolerating the procedure. Other assessment parameters are less critical. Measuring fluid volume intake and output is always important when a client is being mechanically ventilated. Prior attempts at weaning and ECG results are documented on the client's record, and the nurse can refer to them before the weaning process begins

What finding by the nurse may indicate that the patient has chronic hypoxia? a) Cyanosis b) Crackles c) Peripheral edema d) Clubbing of the fingers

D. Clubbing of the fingers is a change in the normal nail bed. It appears as sponginess of the nail bed and loss of the nail bed angle (Fig. 20-6). It is a sign of lung disease that is found in patients with chronic hypoxic conditions, chronic lung infections, or malignancies of the lung (Bickley, 2009). The other signs listed may represent only a temporary hypoxia.

A client undergoes a total laryngectomy and tracheostomy formation. On discharge, the nurse should give which instruction to the client and family? a) "Clean the tracheostomy tube with alcohol and water." b) "Limit the amount of protein in the diet." c) "Oral intake of fluids should be limited for 1 week only." d) "Family members should continue to talk to the client."

D. Commonly, family members are reluctant to talk to the client who has had a total laryngectomy and can no longer speak. To promote a supportive environment, the nurse should encourage family members to continue normal communication. The nurse should teach the client to clean the tracheostomy tube with hydrogen peroxide and rinse it with sterile saline solution, to consume oral fluids as desired, and to eat protein-rich foods to promote healing

A nurse observes constant bubbling in the water-seal chamber of a closed chest drainage system. What should the nurse conclude? a) The client has a pneumothorax. b) The chest tube is obstructed. c) The system is functioning normally. d) The system has an air leak.

D. Constant bubbling in the water-seal chamber indicates an air leak and requires immediate intervention. The client with a pneumothorax will have intermittent bubbling in the water-seal chamber. Clients without a pneumothorax should have no evidence of bubbling in the chamber. If the tube is obstructed, the fluid would stop fluctuating in the water-seal chamber.

Wound drains, inserted during the laryngectomy, stay in place until what criteria are met? a) The stoma is healed, about 6 weeks after surgery. b) The surgical site is dry with encrustations. c) The patient is able to assist with his own suctioning. d) Drainage is <30 mL/day for 2 consecutive days.

D. Drains are removed when secretions are minimal, which usually is less than 30 mL for 48 straight hours.

The nursing instructor is talking with the junior class of nursing students about lung cancer. What would be the best rationale the instructor could give for the difficulty of early diagnosis of lung cancer? a) Symptoms are often minimized by clients. b) There are no early symptoms of lung cancer. c) Symptoms often mimic other infectious diseases. d) Symptoms often do not appear until the disease is well established

D. Early diagnosis of cancer of the lung is difficult because symptoms often do not appear until the disease is well established. Option A is correct, but it is not the best answer. Option B is incorrect because it is not a true statement. Option C is incorrect because lung cancer is not an infectious disease

After lobectomy for lung cancer, a client receives a chest tube connected to a disposable chest drainage system. The nurse observes that the drainage system is functioning correctly when she notes tidal movements or fluctuations in which compartment of the system as the client breathes? a) Air-leak chamber b) Collection chamber c) Suction control chamber d) Water-seal chamber

D. Fluctuations in the water-seal compartment are called tidal movements and indicate normal function of the system as the pressure in the tubing changes with the client's respirations. The air-leak meter — not chamber — detects air leaking from the pleural space. The collection chamber connects the chest tube from the client to the system. Drainage from the tube drains into and collects in a series of calibrated columns in this chamber. The suction control chamber provides the suction, which can be controlled to provide negative pressure to the chest.

The nurse assesses a patient with a heart rate of 42 and a blood pressure of 70/46. What type of hypoxia does the nurse determine this patient is displaying? a) Histotoxic hypoxia b) Anemic hypoxia c) Hypoxic hypoxia d) Circulatory hypoxia

D. Given this patient's vital signs, he appears to be in shock. Circulatory hypoxia is hypoxia resulting from inadequate capillary circulation. It may be caused by decreased cardiac output, local vascular obstruction, low-flow states such as shock, or cardiac arrest. Although tissue partial pressure of oxygen (PO2) is reduced, arterial oxygen (PaO2) remains normal. Circulatory hypoxia is corrected by identifying and treating the underlying cause.

A client with a respiratory condition is receiving oxygen therapy. While assessing the client's PaO2, the nurse knows that the therapy has been effective based on which of the following readings? a) 45 mm Hg b) 58 mm Hg c) 120 mm Hg d) 84 mm Hg

D. In general, clients with respiratory conditions are given oxygen therapy only to increase the arterial oxygen pressure (PaO2) back to the client's normal baseline, which may vary from 60 to 95 mm Hg.

The client you are caring for has just been told they have advanced laryngeal cancer. What is the treatment of choice? a) Laser surgery b) Radiation therapy c) Partial laryngectomy d) Total laryngectomy

D. In more advanced cases, total laryngectomy may be the treatment of choice. Partial laryngectomy, laser surgery, and radiation therapy are not the treatment of choice for advanced cases of laryngeal cancer.

Which of the following clinical manifestations should a nurse monitor for during a pulmonary angiography, which indicates an allergic reaction to the contrast medium? a) Hematoma b) Urge to cough c) Absent distal pulses d) Difficulty in breathing

D. Nurses must determine if the client has any allergies, particularly to iodine, shellfish, or contrast dye. During the procedure, the nurse should check for signs and symptoms of allergic reactions to the contrast medium, such as itching, hives, or difficulty in breathing. The nurses inspects for hematoma, absent distal pulses, after the procedure. When the contrast medium is infused, an urge to cough is often a sensation experienced by the client.

A patient is to receive an oxygen concentration of 70%. What is the best way for the nurse to deliver this concentration? a) A nasal cannula b) A Venturi mask c) An oropharyngeal catheter d) A partial rebreathing mask

D. Partial rebreathing masks have a reservoir bag that must remain inflated during both inspiration and expiration. The nurse adjusts the oxygen flow to ensure that the bag does not collapse during inhalation. A high concentration of oxygen (50% to 75%) can be delivered because both the mask and the bag serve as reservoirs for oxygen. The other devices listed cannot deliver oxygen at such a high concentration.

The nurse is performing chest auscultation for a patient with asthma. How does the nurse describe the high-pitched, sibilant, musical sounds that are heard? a) Rhonchi b) Rales c) Crackles d) Wheezes

D. Sibilant wheezes are continuous, musical, high-pitched, whistlelike sounds heard during inspiration and expiration caused by air passing through narrowed or partially obstructed airways; they may clear with coughing. Crackles, formerly called rales, are soft, high-pitched, discontinuous popping sounds that occur during inspiration (while usually heard on inspiration, they may also be heard on expiration); they may or may not be cleared by coughing. Rhonchi, or sonorous wheezes, are deep, low-pitched rumbling sounds heard primarily during expiration; they are caused by air moving through narrowed tracheobronchial passages.

A patient visited a health care clinic for treatment of upper respiratory tract congestion, fatigue, and sputum production that was rust-colored. Which of the following diagnoses is likely based on this history and inspection of the sputum? a) Bronchitis b) A lung abscess c) Bronchiectasis d) An infection with pneumococcal pneumonia

D. Sputum that is rust colored suggests infection with pneumococcal pneumonia. Bronchiectasis and a lung abscess usually are associated with purulent thick and yellow-green sputum. Bronchitis usually yields a small amount of purulent sputum.

During discharge teaching, a nurse is instructing a client about pneumonia. The client demonstrates his understanding of relapse when he states that he must: a) maintain fluid intake of 40 oz (1,200 ml) per day. b) follow up with the physician in 2 weeks. c) turn and reposition himself every 2 hours. d) continue to take antibiotics for the entire 10 days.

D. The client demonstrates understanding of how to prevent relapse when he states that he must continue taking the antibiotics for the prescribed 10-day course. Although the client should keep the follow-up appointment with the physician and turn and reposition himself frequently, these interventions don't prevent relapse. The client should drink 51 to 101 oz (1,500 to 3,000 ml) per day of clear liquids

A young adult client has had a tonsillectomy and is in the immediate postoperative period. To make the client comfortable, the nurse intervenes by a) Removing the oral airway before the gag reflex has returned for client comfort b) Sitting the client in the semi-Fowler's position c) Maintaining a warm compress around the client's neck area d) Placing the client prone with the head turned to the side

D. The most comfortable position for the client in the immediate postoperative period is prone, not semi-Fowler's. The client's head is turned to the side to allow drainage from the mouth. The oral airway is removed after the gag reflex has returned. An ice collar, not warm compress, is applied to the neck area.

A client is receiving supplemental oxygen. When determining the effectiveness of oxygen therapy, which arterial blood gas value is most important? a) Bicarbonate (HCO3-) b) pH c) Partial pressure of arterial carbon dioxide (PaCO2) d) Partial pressure of arterial oxygen (PaO2)

D. The most significant and direct indicator of the effectiveness of oxygen therapy is the PaO2 value. Based on the PaO2 value, the nurse may adjust the type of oxygen delivery (cannula, Venturi mask, or mechanical ventilator), flow rate, and oxygen percentage. The other options reflect the client's ventilation status, not oxygenation. The pH, HCO3-, and PaCO2

A client is prescribed two sprays of a nasal medication twice a day. The nurse is teaching the client how to self-administer the medication and instructs the client to a) Tilt the head back when activating the spray of the medication. b) Wait 10 seconds before administering the second spray. c) Clean the medication container once each day. d) Blow the nose before applying medication into the nares.

D. The nurse instructs the client to blow the nose before administering the nasal medication. The client should keep the head upright, not tilted back. The client should wait at least 1 minute before administering the second spray and clean the container after each use

The nurse is caring for a client with allergic rhinitis. The patient asks the nurse about measures to help decrease allergic symptoms. The best response by the nurse is which of the following? A. Get your influenza vaccination yearly B. Take OTC nasal decongestants C. Refer to an ENT physician D. avoid exposure to allergens and irritants

D. The nurse instructs the patient with allergic rhinitis to avoid or reduce exposure to allergens and irritants, such as dusts, molds, animals, fumes, odors, powders, sprays, and tobacco smoke. Receiving an influenza vaccination each year is recommended for patients with infectious rhinitis. To prevent possible drug interactions, the patient is cautioned to read drug labels before taking any OTC medication. Patients with nasal septal deformities or nasal polyps may be referred to an ear, nose, and throat specialist.

The nurse assessed a 28-year-old woman who was experiencing dyspnea severe enough to make her seek medical attention. The history revealed no prior cardiac problems and the presence of symptoms for 6 months' duration. On assessment, the nurse noted the presence of both inspiratory and expiratory wheezing. Based on this data, which of the following diagnoses is likely? a) Acute respiratory obstruction b) Pneumothorax c) Adult respiratory distress syndrome d) Asthma

D. The presence of both inspiratory and expiratory wheezing usually signifies asthma if the individual does not have heart failure. Sudden dyspnea is an indicator of the other choices

In general, chest drainage tubes are not used for the patient undergoing a) wedge resection. b) lobectomy. c) segmentectomy. d) pneumonectomy.

D. Usually, no drains are used for the pneumonectomy patient because the accumulation of fluid in the empty hemothorax prevents mediastinal shift. With lobectomy, two chest tubes are usually inserted for drainage, the upper tube for air and the lower tube for fluid. With wedge resection, the pleural cavity usually is drained because of the possibility of an air or blood leak. With segmentectomy, drains are usually used because of the possibility of an air or blood leak.

What is the difference between respiration and ventilation? a) Ventilation is the exchange of gases in the lung. b) Ventilation is the process of gas exchange. c) Ventilation is the process of getting oxygen to the cells. d) Ventilation is the movement of air in and out of the respiratory tract.

D. Ventilation is the actual movement of air in and out of the respiratory tract. Respiration is the exchange of oxygen and CO2 between atmospheric air and the blood and between the blood and the cells. Therefore, options A, B, and C are incorrect.

When performing endotracheal suctioning, the nurse applies suctioning while withdrawing and gently rotating the catheter 360 degrees for which of the following time periods? a) 10 to 15 seconds b) 20 to 25 seconds c) 0 to 5 seconds d) 30 to 35 seconds

A. In general, the nurse should apply suction no longer than 10 to 15 seconds because hypoxia and dysrhythmias may develop, leading to cardiac arrest. Applying suction for 30 to 35 seconds is hazardous and may result in the patient's developing hypoxia, which can lead to dysrhythmias and, ultimately, cardiac arrest. Applying suction for 20 to 25 seconds is hazardous and may result in the patient's developing hypoxia, which can lead to dysrhythmias and, ultimately, cardiac arrest. Applying suction for 0 to 5 seconds would provide too little time for effective suctioning of secretions.

A client seeks care for hoarseness that has lasted for 1 month. To elicit the most appropriate information about this problem, the nurse should ask which question? a) "Do you smoke cigarettes, cigars, or a pipe?" b) "Do you eat a lot of red meat?" c) "Do you eat spicy foods?" d) "Have you strained your voice recently?"

A. Persistent hoarseness may signal throat cancer, which commonly is associated with tobacco use. To assess the client's risk for throat cancer, the nurse should ask about smoking habits. Although straining the voice may cause hoarseness, it wouldn't cause hoarseness lasting for 1 month. Consuming red meat or spicy foods isn't associated with persistent hoarseness

Pink frothy sputum may be an indication of a) pulmonary edema. b) bronchiectasis. c) an infection. d) a lung abscess.

A. Profuse, frothy pink material, often welling up into the throat, may indicate pulmonary edema. Foul-smelling sputum and bad breath may indicate a lung abscess, bronchiectasis, or an infection caused by fusospirochetal or other anaerobic organisms

A client with thrombocytopenia, secondary to leukemia, develops epistaxis. The nurse should instruct the client to: a) sit upright, leaning slightly forward. b) blow his nose and then put lateral pressure on his nose. c) hold his nose while bending forward at the waist. d) lie supine with his neck extended.

A. Sitting upright and leaning slightly forward avoids increasing vascular pressure in the nose and helps the client avoid aspirating blood. Lying supine won't prevent aspiration of the blood. Nose blowing can dislodge any clotting that has occurred. Bending at the waist increases vascular pressure in the nose and promotes bleeding rather than halting it.

Which of the following would not be considered a primary symptom of COPD? a) Dyspnea on exertion b) Weight gain c) Cough d) Sputum production

B. COPD is characterized by three primary symptoms: cough, sputum production, and dyspnea on exertion. Weight loss is common with COPD.

The nurse receives an order to obtain a sputum sample from a client with hemoptysis. When advising the client of the physician's order, the client states not being able to produce sputum. Which suggestion, offered by the nurse, is helpful in producing the sputum sample? a) Drink 8 oz of water to thin the secretions for expectoration. b) Take deep breaths and cough forcefully. c) Use the secretions present in the oral cavity. d) Tickle the back of the throat to produce the gag reflex

B. Taking deep breaths moves air around the sputum and coughing forcefully moves the sputum up the respiratory tract. Once in the pharynx, the sputum can be expectorated into a specimen container. Producing a gag reflex elicits stomach contents and not respiratory sputum. Dilute and thinned secretions are not helpful in aiding expectoration. A sputum culture is not a component of oral secretions.

A client with chronic obstructive pulmonary disease (COPD) is intubated and placed on continuous mechanical ventilation. Which equipment is most important for the nurse to keep at this client's bedside? A. Oxygen analyzer B. Manual resuscitation bag C. Water-seal chest drainage set up D. Tracheostomy cleaning kit

B. The client with COPD depends on mechanical ventilation for adequate tissue oxygenation. The nurse must keep a manual resuscitation bag at the bedside to ventilate and oxygenate the client in case the mechanical ventilator malfunctions. Because the client doesn't have chest tubes or a tracheostomy, keeping a water-seal chest drainage set-up or a tracheostomy cleaning kit at the bedside isn't necessary. Although the nurse may keep an oxygen analyzer (pulse oximeter) on hand to evaluate the effectiveness of ventilation, this equipment is less important than the manual resuscitation bag

Of the following oxygen administration devices, which has the advantage of providing high oxygen concentration? a) Face tent b) Nonrebreather mask c) Venturi mask d) Catheter

B. The nonrebreather mask provides high oxygen concentration, but it is usually poorly fitting. However, if the nonrebreathing mask fits the patient snugly and both side exhalation ports have one-way valves, it is possible for the patient to receive 100% oxygen, making the nonrebreathing mask a high-flow oxygen system. The Venturi mask provides low levels of supplemental oxygen. The catheter is an inexpensive device that provides a variable fraction of inspired oxygen and may cause gastric distention. A face tent provides a fairly accurate fraction of inspired oxygen but is bulky and uncomfortable. It would not be the device of choice to provide high oxygen concentration

A client experiences a head injury in a motor vehicle accident. The client's level of consciousness is declining, and respirations have become slow and shallow. When monitoring a client's respiratory status, which area of the brain would the nurse realize is responsible for the rate and depth? a) Central sulcus b) The pons c) Wernicke's area d) The frontal lobe

B. The pons in the brainstem controls rate and depth of respirations. When injury occurs or increased intracranial pressure results, respirations are slowed. The frontal lobe completes executive functions and cognition. The central sulcus is a fold in the cerebral cortex called the central fissure. The Wernicke's area is the area linked to speech

The term for the volume of air inhaled and exhaled with each breath is: A. Expiratory reserve volume B. Tidal volume C. Residual volume D. Vital capacity

B. Tidal volume is the volume of air inhaled and exhaled with each breath. Residual volume is the volume of air remaining in the lungs after a maximum expiration. Vital capacity is the maximum volume of air exhaled from the point of maximum inspiration. Expiratory reserve volume is the maximum volume of air that can be exhaled after a normal inhalation.

The nurse is assessing a patient who has been admitted with possible ARDS. What findings would distinguish ARDS from cardiogenic pulmonary edema? a) Elevated troponin levels b) Elevated white blood count c) Elevated B-type natriuretic peptide (BNP) levels d) Elevated myoglobin levels

C. Common diagnostic tests performed in patients with potential ARDS include plasma brain natriuretic peptide (BNP) levels, echocardiography, and pulmonary artery catheterization. The BNP level is helpful in distinguishing ARDS from cardiogenic pulmonary edema

Lung compliance (the ability of the lungs to stretch) is a physical factor that affects ventilation. A nurse is aware that a patient who has lost elasticity in the lung tissue has a condition known as: a) Pulmonary edema b) Atelectasis c) Emphysema d) Pleural effusion

C. Emphysema, most commonly caused by smoking cigarettes, results in a loss of lung elasticity, which destroys the capillaries that supply the alveoli. This causes airway collapse during expiration, dyspnea, and eventually cyanosis

What dietary recommendations should a nurse provide a patient with a lung abscess? a) A diet with limited fat b) A carbohydrate-dense diet c) A diet rich in protein d) A diet low in calories

C. For a patient with pleural effusion, a diet rich in protein and calories is pivotal. A carbohydrate-dense diet or diets with limited fat are not advisable for a patient with lung abscess

The nurse knows that a sputum culture is necessary to identify the causative organism for acute tracheobronchitis. What causative fungal organism would the nurse suspect? a) Haemophilus b) Mycoplasma pneumoniae c) Aspergillus d) Streptococcus pneumoniae

C. In acute tracheobronchitis, the inflamed mucosa of the bronchi produces mucopurulent sputum, often in response to infection by Streptococcus pneumoniae, Haemophilus influenzae, or Mycoplasma pneumoniae. In addition, a fungal infection (e.g., Aspergillus) may also cause tracheobronchitis. A sputum culture is essential to identify the specific causative organism.

A client with myasthenia gravis is receiving continuous mechanical ventilation. When the high-pressure alarm on the ventilator sounds, what should the nurse do? a) Check for an apical pulse. b) Increase the oxygen percentage. c) Ventilate the client with a handheld mechanical ventilator. d) Suction the client's artificial airway

D. A high-pressure alarm on a continuous mechanical ventilator indicates an obstruction in the flow of oxygen from the machine to the client. The nurse should suction the client's artificial airway to remove respiratory secretions that could be causing the obstruction. The sounding of a ventilator alarm has no relationship to the apical pulse. Increasing the oxygen percentage and ventilating with a handheld mechanical ventilator wouldn't correct the airflow blockage

The nurse is assessing the lungs of a patient diagnosed with pulmonary edema. Which of the following would be expected upon auscultation? a) Egophony b) Bronchial breath sounds c) Absent breath sounds d) Crackles at lung bases

D. A patient with pulmonary edema would be expected to have crackles in the lung bases, and possible wheezes. Egophony may occur in patients diagnosed with pleural effusion. Absent breath sounds occurs in pneumothorax. Bronchial breath sounds occur in consolidation, such as pneumonia.

A client has a nursing diagnosis of "ineffective airway clearance" as a result of excessive secretions. An appropriate outcome for this client would be which of the following? a) Client reports no chest pain. b) Respiratory rate is 12 to 18 breaths per minute. c) Client can perform incentive spirometry. d) Lungs are clear on auscultation.

D. Assessment of lung sounds includes auscultation for airflow through the bronchial tree. The nurse evaluates for fluid or solid obstruction in the lung. When airflow is decreased, as with fluid or secretions, adventitious sounds may be auscultated. Often crackles are heard with fluid in the airways.

Which of the following is a true statement regarding severe acute respiratory syndrome (SARS)? a) Hypothermia will occur b) It is spread by fecal contamination c) Constipation usually develops d) It is the most contagious during the second week of illness

D. Based on available information, SARS is most likely to be contagious only when symptoms are present, and patients are most contagious during the second week of illness. Diarrhea and hyperthermia may occur with SARS. Respiratory droplets spread the SARS virus when an infected person coughs or sneezes

A physician stated to the nurse that the patient has fluid noted in the pleural space and will need a thoracentesis. The nurse would expect that the physician will document this fluid as which of the following? A. Hemothorax B. Consolidation C. Pneumothorax D. Pleural effusion

D. Fluid accumulating within the pleural space is called a pleural effusion. A pneumothorax is air in the pleural space. A hemothorax is blood within the pleural space. Consolidation is lung tissue that has become more solid in nature due to collapse of alveoli or infectious process.

The herpes simplex virus type 1 (HSV-1), which produces a cold sore (fever blister), has an incubation period of a) 3 to 6 months. b) 20 to 30 days. c) 0 to 3 months. d) 2 to12 days.

D. HSV-1 is transmitted primarily by direct contact with infected secretions. The time period 0 to 3 months exceeds the incubation period. The time period 20 to 30 days exceeds the incubation period. The time period 3 to 6 months exceeds the incubation period

A nurse is caring for a client who was intubated because of respiratory failure. The client is now receiving mechanical ventilation with a preset tidal volume and number of breaths each minute. The client has the ability to breathe spontaneously between the ventilator breaths with no ventilator assistance. The nurse should document the ventilator setting as: a) assist-control (AC) ventilation. b) pressure support ventilation (PSV). c) continuous positive airway pressure (CPAP). d) synchronized intermittent mandatory ventilation (SIMV).

D. In SIMV mode, the ventilator delivers a preset number of breaths at a preset tidal volume. The client can breathe on his own in between the breaths delivered by the ventilator. In PSV, a pressure plateau is added to the ventilator to prevent the airway pressure from falling beneath a preset level. In AC ventilation, the ventilator delivers a preset number of breaths at a preset tidal volume and any breaths that the client takes on his own are assisted by the ventilator so they reach the preset tidal volume. In CPAP, the ventilator provides only positive airway pressure; it doesn't provide any breaths to the client.

Histamine, a mediator that supports the inflammatory process in asthma, is secreted by a) Eosiniphils b) Neutrophils c) Lymphocytes d) Mast cells

D. Mast cells, neutrophils, eosinophils, and lymphocytes play key roles in the inflammation associated with asthma. When activated, mast cells release several chemicals called mediators. One of these chemicals is called histamine

The nurse is caring for a client with chronic obstructive pulmonary disease. The client calls the doctor and states having difficulty breathing and overall feeling fatigued. The nurse realizes that this client is at high risk for which condition? a) Metabolic acidosis b) Respiratory alkalosis c) Metabolic alkalosis d) Respiratory acidosis

D. Respiratory acidosis occurs when the body is unable to blow off CO2 due to the hypoventilation of disease processes such as COPD. An increase in blood carbon dioxide concentration occurs and a decreased pH causing acidosis. Respiratory alkalosis is a decrease in acidity of the blood and often caused by hyperventilation. Metabolic acidosis/alkalosis are disorders that affect the bicarbonate

A patient has a Mantoux skin test prior to being placed on an immunosuppressant for the treatment of Crohn's disease. What results would the nurse determine is not significant for holding the medication? a) 7 to 8 mm b) 9 mm c) 5 to 6 mm d) 0 to 4 mm

D. The Mantoux method is used to determine whether a person has been infected with the TB bacillus and is used widely in screening for latent M. tuberculosis infection. The size of the induration determines the significance of the reaction. A reaction of 0 to 4 mm is considered not significant. A reaction of 5 mm or greater may be significant in people who are considered to be at risk

A son brings his father into the clinic, stating that his father's color has changed to bluish around the mouth. The father is confused, with a respiratory rate of 28 breaths per minute and scattered crackles throughout. The son states this condition just occurred within the last hour. Which of the following factors indicates that the client's condition has lasted for more than 1 hour? a) Crackles b) Respiratory rate c) Son's statement d) Cyanosis

D. The client's appearance may give clues to respiratory status. Cyanosis, a bluish coloring of the skin, is a very late indicator of hypoxia. The presence of cyanosis is from decreased unoxygenated hemoglobin. In the presence of a pulmonary condition, cyanosis is assessed by observing the color of the tongue and lips

Which of the following is true about both lung transplant and bullectomy? a) Both are aimed at curing COPD. b) Both are aimed at treating end-stage emphysema. c) Both are used to treat patients with bullous emphysema. d) Both are aimed at improving the overall quality of life of a patient with COPD.

D. The treatments for COPD are aimed more at treating the symptoms and preventing complications, thereby improving the overall quality of life of a patient with COPD. In fact, there is no cure for COPD. Lung transplant is aimed at treating end-stage emphysema and bullectomy is used to treat patients with bullous emphysema

A physician has ordered that a client with suspected lung cancer undergo magnetic resonance imaging (MRI). The nurse explains the benefits of this study to the client. Included in teaching would be which of the following regarding the MRI? a) Lung blood flow can be viewed after a radiopaque agent is injected. b) MRI can view soft tissues and can help stage cancers. c) Narrow-beam x-ray can scan successive lung layers. d) Tumor densities can be seen with radiolucent images.

B. MRI uses magnetic fields and radiofrequency signals to produce a detailed diagnostic image. MRI can visualize soft tissues, characterize nodules, and help stage carcinomas. The other options describe different studies

The classification of Stage I of COPD is defined as a) severe COPD. b) at risk for COPD. c) mild COPD. d) very severe COPD.

C. Stage I is mild COPD. Stage 0 is at risk for COPD. Stage III is severe COPD. Stage IV is very severe COPD

A client arrives at the physician's office stating dyspnea; a productive cough for thick, green sputum; respirations of 28 breaths/minute, and a temperature of 102.8° F. The nurse auscultates the lung fields, which reveal poor air exchange in the right middle lobe. The nurse suspects a right middle lobe pneumonia. To be consistent with this anticipated diagnosis, which sound, heard over the chest wall when percussing, is anticipated? a) Tympanic b) Resonant c) Hyperresonant d) Dull

D. a dull percussed sound, heard over the chest wall, is indicative of little or no air movement in that area of the lung. Lung consolidation such as in pneumonia or fluid accumulation produces the dull sound. A tympanic sound is a high-pitched sound commonly heard over the stomach or bowel. A resonant sound is noted over normal lung tissue. A hyper resonant sound is an abnormal lower pitched sound that occurs when free air exists in disease processes such as pneumothorax

After reviewing the pharmacological treatment for pulmonary diseases, the nursing student knows that bronchodilators relieve bronchospasm in three ways. Choose the correct three of the following options. a) Reduce airway obstruction b) Alter smooth muscle tone c) Decrease alveolar ventilation d) Increase oxygen distribution

A, B, D. Bronchodilators relieve bronchospasm by altering smooth muscle tone and reduce airway obstruction by allowing increased oxygen distribution throughout the lungs and improving alveolar ventilation

A client has a nursing diagnosis of acute pain related to upper airway irritation. The best short-term goal for this client is for the client to a) Report relief of pain to level 3 using a pain intensity scale of 1 to 10. b) Use a pain intensity rating scale of 0 to 10. c) Gargle with a warm saline solution frequently. d) Take acetaminophen with codeine when pain is 5 or above

A. The client statement of relief of pain to level 3 indicates improvement of the problem. The other options are actually interventions or actions that can help achieve a long-term goal of relief of pain

A nurse understands that a safe but low level of oxygen saturation provides for adequate tissue saturation while allowing no reserve for situations that threaten ventilation. What is a safe but low oxygen saturation level for a patient? a) 40% b) 95% c) 75% d) 80%

B. With a normal value for the partial pressure of oxygen (PaO2) (80 to 100 mm Hg) and oxygen saturation (SaO2) (95% to 98%), there is a 15% margin of excess oxygen available to the tissues. With a normal hemoglobin level of 15 mg/dL and a PaO2 level of 40 mm Hg (SaO2 75%), there is adequate oxygen available for the tissues but no reserve for physiologic stresses that increase tissue oxygen demand.

The nurse is assisting a physician with an endotracheal intubation for a client in respiratory failure. It is most important for the nurse to assess for: a) Cool air humidified through the tube b) Tracheal cuff pressure set at 30 mm Hg c) Symmetry of the client's chest expansion d) A scheduled time for deflation of the tracheal cuff

C. Immediately after intubation, the nurse should check for symmetry of chest expansion. This is one finding that indicates successful endotracheal placement. The tracheal cuff pressure is set between 15 and 20 mm Hg. Routine deflation of the tracheal cuff is not recommended, because the client could then aspirate secretions during the period of deflation. Warm, high, humidified air is administered through the endotracheal tube

The nurse is educating a patient with COPD about the technique for performing pursed-lip breathing. What does the nurse inform the patient is the importance of using this technique? a) It will prevent the alveoli from overexpanding. b) It increases the respiratory rate to improve oxygenation. c) It prolongs exhalation. d) It will assist with widening the airway.

C. The goal of pursed-lip breathing is to prolong exhalation and increase airway pressure during expiration, thus reducing the amount of trapped air and the amount of airway resistance

The nurse is caring for a client experiencing laryngeal trauma. Upon assessment, swelling and bruising is noted to the neck. Which breath sound is anticipated? a) Rhonchi in the bronchial region b) Diminished breath sounds throughout c) Audible stridor without using a stethoscope d) Crackles in the bases of the lungs

C. The nurse anticipates hearing audible stridor without needing a stethoscope due to the neck swelling narrowing the airway. Rhonchi in the bronchial region is heard lower in the airways and crackles are heard in the bases of the lungs. Diminished breath sounds that occur throughout are indicative of airway obstruction and not indicative of laryngeal swelling.

Which of the following clinical manifestations of hemorrhage is related to carotid artery rupture? a) Increased blood pressure b) Shallow respirations c) Increased pulse rate d) Dry skin

C. The nurse monitors vital signs for changes, particularly increased pulse rate, decreased blood pressure, and rapid, deep respirations. Cold, clammy, pale skin may indicate active bleeding.

Which of the following interventions does a nurse implement for patients with empyema? a) Institute droplet precautions b) Place suspected patients together c) Encourage breathing exercises d) Do not allow visitors with respiratory infection

C. The nurse teaches the patient with empyema to do breathing exercises as prescribed. The nurse should institute droplet precautions and isolate suspected and confirmed influenza patients in private rooms or place suspected and confirmed patients together. The nurse does not allow visitors with symptoms of respiratory infection to visit the hospital to prevent outbreaks of influenza from occurring in health care settings

The nurse is preparing to perform chest physiotherapy (CPT) on a patient. Which of the following patient statements would indicate the procedure is contraindicated. a) "I received my pain medication 10 minutes ago, let's do my CPT now." b) "I just changed into my running suit; we can do my CPT now." c) "I just finished eating my lunch, I'm ready for my CPT now." d) "I have been coughing all morning and am barely bringing anything up."

C. When performing CPT, the nurse ensures that the patient is comfortable, is not wearing restrictive clothing, and has not just eaten. The nurse gives medication for pain, as prescribed, before percussion and vibration and splints any incision and provides pillows for support, as needed. A goal of CPT is for the patient to be able to mobilize secretions; the patient who is having an unproductive cough is a candidate for CPT.

The nurse is reviewing the blood gas results for a patient with pneumonia. What arterial blood gas measurement best reflects the adequacy of alveolar ventilation? a) SaO2 b) pH c) PaCO2 d) PaO2

C. When the minute ventilation falls, alveolar ventilation in the lungs also decreases, and the PaCO2 increases.


Kaugnay na mga set ng pag-aaral

A&P - Nutrition, Metabolism, and Energy Balance

View Set

SAS Base Programming Review - Numeric Answers Only

View Set

Review Sheet Drugs and Diagnosis

View Set